2
競技数学解説
文献あり

ISL2018-A3

268
0
$$$$

はじめに

 最近解いたISLの問題がかなり面白かった(のと夏休みの宿題が溜まった)ので,その問題の解説をしようと思います.

(ISL2018-A3 原文をかなり適当に翻訳しました)

 任意の正の整数からなる集合$S$について,次のうち少なくとも$1$つが成り立つことを示せ.

$\mathbf{(A)}$ $S$のある部分集合$F,G ~ (F \neq G)$が存在して,$\displaystyle \sum_{x \in F} \frac{1}{x}=\sum_{x \in G} \frac{1}{x}$

$\mathbf{(B)}$ ある$1$未満の正の有理数$r$が存在し,任意の$S$の部分集合$F$について,$\displaystyle \sum_{x \in F} \frac{1}{x} \neq r$

 ネタバレを含みます.あと原文では条件が$(1)$$(2)$になっていますが見やすさのためにこうしてます.ご了承下さい...

.
..
.
...
.
..
.
....
.
..
.
...
.
..
.

思ったこと

 かなり主張が綺麗な感じですね.というかそもそも$\mathbf{(B)}$を満たさない$S$が思い付きません(多分正整数全体の集合は満たすんだろうけど証明ができない).多分直接示すのは難しそうです.

考えたこと

すぐ分かること

 少なくとも$1$つが成り立つ、は流石にやばいです.どっちも成り立たない集合$S$を考えてあげます.

(ISL2018-A3 改)

 正の整数からなる集合$S$であって,次をともに満たすものは存在しないことを示せ.

$\mathbf{(A^*)}$ $S$の任意の異なる部分集合$F,G$について,$\displaystyle \sum_{x \in F} \frac{1}{x} \neq \sum_{x \in G} \frac{1}{x}$

$\mathbf{(B^*)}$ 任意の$1$未満の正の有理数$r$について,ある$S$の部分集合$F$が存在し,$\displaystyle \sum_{x \in F} \frac{1}{x} = r$

 この条件を言語化してみると,「正の有理数を逆数和で表す方法はせいぜい$1$通りだよね、特に$1$未満なら必ず$1$個あるよね、(謎口調」となります.$S$$2$進法の進化版みたいな,スーパーハイパーなめっちゃ強いやつというイメージです.${}_{(??)}$

 まず,$1$未満の正の有理数は無限に存在するので,$S$は無限集合だと分かります.ちなみにですが,この考察があまりにも明らかだからといって,答案に書き忘れないようにしましょう.($1$敗)

 また,$S$$1$を含まないとします.$1$が満たさないことが分かったら,$1$があっても満たしません.まあどうせ$1$はそんなに関係ないので...

次に分かること

 $r$は常に逆数和で表せることが決まっているので,$r+\dfrac{1}{y} ~ (\lt 1)$のような形を考えると何かできそうです.$r$を逆数和で表すとき,$1/y$が現れないとしてみます.すると,$r+\dfrac{1}{y}$には$1/y$が現れます(当たり前ですが).逆のこともいえると嬉しいです.つまり

$r$を逆数和で表すとき$1/y$が現れるなら,$r+\dfrac{1}{y}$には$1/y$が現れない.

が言えて欲しいです.これを示してみます.もし$r+\dfrac{1}{y}$$1/y$が現れるなら,$r$には$1/y$が現れません.ですがこれは前提と$\mathbf{(A^*)}$に矛盾しています.よって示されました.また,対偶をとることで

$r$を逆数和で表すとき$1/y$が現れないなら,$r-\dfrac{1}{y} ~ (\gt 0)$には$1/y$が現れる.

$r$を逆数和で表すとき$1/y$が現れるなら,$r-\dfrac{1}{y} ~ (\gt 0)$には$1/y$が現れない.

が分かります.

 ...ここで,この条件はめちゃくちゃ強いということが分かります.というのも

$r-\dfrac{1}{y} ~ (\gt 0)$には$1/y$が現れる $→$ $r \geq \dfrac{2}{y}$

だからです.そこで,明らかに$1/y$が現れず,$1/y$より大きい$r$を取ってくると良さそうです.これはいろいろ考えると,$y$$1$つ前の要素(小さい順に並べたとき.)の逆数がはまるのでこれを$1/z$とすると,

$\dfrac{1}{z} \geq \dfrac{2}{y} →$ $z \leq 2y$

が分かります.つまり,$S$のうち$k$番目に小さい要素は$2^k$で下から抑えられることが分かりました.$2^k$と聞いてピンときた人もいるかもしれませんが,これらの逆数和は$1$以下の値に収束します! もし$1$未満の値に収束したら,それより大きく$1$より小さい有理数を$r$にすれば$\mathbf{(B^*)}$に合いません.なので,$1$に収束し,$S=\{2,4,8, \ldots 2^k, \ldots \}$ ですが,このとき$\dfrac{1}{3}$などは(有限個の)逆数和になりません.よって,この問題が解けました!

 もうちょっと正確に議論する必要がありそうですが,一応これで解けています.答案では見やすいように文字の置き方とかを変えています.(なら上でもその置き方にすべきでは)

答案

 $S$が問題の条件を満たすなら,$\{1\} \cup S$も条件を満たす.以下では$1 \notin S$とする.次をともに満たす$S$が存在すると仮定する.この$S$は,明らかに無限集合である.

$\mathbf{(A^*)}$ $S$の任意の異なる部分集合$F,G$について,$\displaystyle \sum_{x \in F} \frac{1}{x} \neq \sum_{x \in G} \frac{1}{x}$

$\mathbf{(B^*)}$ 任意の$1$未満の正の有理数$r$について,ある$S$の部分集合$F$が存在し,$\displaystyle \sum_{x \in F} \frac{1}{x} = r$

 $\mathbf{(B^*)}$$r$について,そのような$F$$r$$\textbf{表現}$とよぶ.また,$S$の要素のうち$k$番目に小さいものを$s_k$とする.

 今,$\dfrac{1}{s_k}-\dfrac{1}{s_{k+1}}$の表現には$s_{k+1}$が含まれる.なぜなら,含まれないと仮定したとき,$\bigg(\dfrac{1}{s_{k}}-\dfrac{1}{s_{k+1}}\bigg)+\dfrac{1}{s_{k+1}}=\dfrac{1}{s_k}$$s_{k+1}$を含む表現が存在するが,これは$\mathbf{(A^*)}$に矛盾するからである.特に

$$\dfrac{1}{s_{k}}-\dfrac{1}{s_{k+1}} \geq \dfrac{1}{s_{k+1}} ~ → ~ 2s_k \leq s_{k+1}$$
であり,$2 \leq s_1$より帰納的に$2^k \leq s_k$が分かる.

 $s_i \gt 2^i$となる$i$が存在するとき,$\displaystyle \sum_{k=1}^{\infty} \dfrac{1}{s_k} \lt \sum_{k=1}^{\infty} \dfrac{1}{2^k} = 1$より$\displaystyle \sum_{k=1}^{\infty} \dfrac{1}{s_k}$$1$未満の正の実数に収束するが,この収束値より大きく$1$より小さい有理数を$r$としてとれば$\mathbf{(B^*)}$を満たさない.
 従って,任意の$k$について$s_k=2^k$だが,このとき$S$の有限個の要素の逆数和として表せる有理数は,既約分数で表したとき分母が偶数であるから,$r=\dfrac{1}{2023}$などをとれば$\mathbf{(B^*)}$は満たされない.

 以上より,$\mathbf{(A^*)},\mathbf{(B^*)}$をともに満たす$S$は存在せず,題意は示された.■

感想

 こういうタイプの代数はかなり好きです.A3にしては易しめな気はしますが...

参考文献

投稿日:202388

この記事を高評価した人

高評価したユーザはいません

この記事に送られたバッジ

バッジはありません。

投稿者

じゃむ
じゃむ
27
1848
競技数学を食べています

コメント

他の人のコメント

コメントはありません。
読み込み中...
読み込み中